La ricerca ha trovato 77 risultati

da bern-1-16-4-13
11 feb 2016, 21:38
Forum: Combinatoria
Argomento: Bello
Risposte: 7
Visite : 4067

Re: Bello

Ora mi viene il dubbio: la tua domanda è rivolta a cip vero?
da bern-1-16-4-13
10 feb 2016, 20:16
Forum: Combinatoria
Argomento: $n$ ragazze e $2n-1$ ragazzi
Risposte: 4
Visite : 2648

Re: $n$ ragazze e $2n-1$ ragazzi

LEMMA:
$$\left(\frac{n!}{\left(n-r\right)!}\right)^2=r\sum_{k=r}^n\left(\left(\frac{\left(k-1\right)!}{\left(k-r\right)!}\right)^{2}\left(2k-r\right)\right)\ \ \ \ \ \forall r\leq n:\ \ r,n\in\mathbb{Z}^+.$$

Come si dimostra? :roll:
da bern-1-16-4-13
10 feb 2016, 20:13
Forum: Combinatoria
Argomento: Bello
Risposte: 7
Visite : 4067

Re: Bello

Se non sbaglio nel momento in cui uno vede che due cicli dispari non possono avere più di 1 vertice in comune, e che non si può avere un "ciclo" di cicli dispari (se rappresentiamo con un punto ogni ciclo dispari e con un arco tra due vertici il fatto che i rispettivi cicli abbiano un vert...
da bern-1-16-4-13
03 feb 2016, 22:01
Forum: Olimpiadi della matematica
Argomento: Winter Camp 2016
Risposte: 134
Visite : 59551

Re: Winter Camp 2016

- Bernardo che, volendo perdere per l'ultima volta a subotto contro Gori e dovendo andare al cesso ed essendo affetto dalla compulsione di dimenticarsi lo spazzolino, fa perdere il treno ad entrambi... <3 <3 <3 L'emozione di "correre" (forse nel tuo caso è più corretto dire "aumentar...
da bern-1-16-4-13
03 feb 2016, 20:35
Forum: Combinatoria
Argomento: Ma che risultato simpatico!
Risposte: 4
Visite : 3056

Re: Ma che risultato simpatico!

Però formulato così sembrerebbe che $n$ non dipenda da $m$...

EDIT:
Gerald Lambeau ha scritto:da un insieme fissato di m primi distinti
No va be', lascia stare, sono io che sono cieco e non so leggere i testi dei problemi
da bern-1-16-4-13
24 gen 2016, 01:17
Forum: Teoria dei Numeri
Argomento: Coprimi
Risposte: 4
Visite : 3110

Re: Coprimi

D'accordo, ma quando ho visto che quel passo base veniva (abbastanza facilmente, si tratta solo di fare i calcoli) con i generatori modulo $p^k$ non potevo non usarli!! :lol:
da bern-1-16-4-13
24 gen 2016, 00:38
Forum: Teoria dei Numeri
Argomento: Coprimi
Risposte: 4
Visite : 3110

Re: Coprimi

Alla fine hai fatto tutti questi giri di parole con funzioni per chiedere di dimostrare un lemma!! :lol: Lemma: $$\prod_{t\in T_n}t\equiv 1\pmod{n}\ \ \ \forall n\in\mathbb{N}.$$ Se la sommatoria è vuota allora si presume sia uguale a $1$, quindi per $n\equiv 0\pmod{2}$ la tesi è banalmente vera (qu...
da bern-1-16-4-13
22 gen 2016, 17:17
Forum: Teoria dei Numeri
Argomento: 195. Divisibilità
Risposte: 4
Visite : 3018

Re: 195. Divisibilità

cip999 ha scritto: Invece, per i bravi ragazzi che non usano cose illegali come i generatori modulo $p^k$
Quindi io sarei un ragazzo cattivo e tu uno bravo?! :x
Ahahahah, nemmeno avessi utilizzato bombe atomiche tipo la congettura di Bunyakovsky :lol:
da bern-1-16-4-13
22 gen 2016, 15:25
Forum: Teoria dei Numeri
Argomento: 195. Divisibilità
Risposte: 4
Visite : 3018

Re: 195. Divisibilità

FATTO1: diciamo che $4^n-1=3k$ (ovviamente $k\in\mathbb{N}$). Allora $$\exists\ \ \ \ \mathbb{P}\ni p\equiv 3\pmod{4}:\ \ p\mid k\Longleftrightarrow\nexists \ \ \ \ j\in\mathbb{N}:\ \ n=2^j.$$ Dimostrazione freccia "$\Longleftarrow$": se $n$ non è potenza di $2$ allora $n=2^{v_2\left(n\ri...
da bern-1-16-4-13
21 gen 2016, 00:27
Forum: Algebra
Argomento: AMC 1984 modificata
Risposte: 6
Visite : 3666

Re: AMC 1984 modificata

Let's start... Supponiamo l'assurdo. Chiamiamo $k$ un numero naturale diverso da $0$ e $1$ (si ottiene facilmente che $f\left(0\right)=0,f\left(1\right)=1$) per cui $f\left(k\right)\neq k$. Allora si deve ovviamente avere che $f\left(k\right)<k\vee f\left(k\right)>k$. Analizzo solo il primo caso poi...
da bern-1-16-4-13
07 gen 2016, 18:41
Forum: Olimpiadi della matematica
Argomento: Winter Camp 2016
Risposte: 134
Visite : 59551

Re: Winter Camp 2016

Quanti problemi avete dovuto correggere in 6 giorni?? (=quante sono state le domande di ammissione)
da bern-1-16-4-13
31 dic 2015, 10:27
Forum: Algebra
Argomento: Somma di quadrati
Risposte: 5
Visite : 3834

Re: Somma di quadrati

Se poniamo \frac{x}{1}=a,\ \ \frac{y}{x}=b,\ \ \frac{z}{y}=c,\ \ \frac{4}{z}=d allora abcd=4 . Vogliamo quindi trovare il minimo di \sum_{cyc}\left(a-1\right)^2 . Ma per QM-AM fissata la media aritmetica, la media quadratica è minima se a-1=b-1=c-1=d-1 quindi se a=b=c=d . Ma noi conosciamo la media ...
da bern-1-16-4-13
16 dic 2015, 22:21
Forum: Olimpiadi della matematica
Argomento: Winter Camp 2016
Risposte: 134
Visite : 59551

Re: Winter Camp 2016

Macché finita!!! Non s'abbia a fare troppi giorni di vacanza! Sennò poi ci si dimentica materie basilari come il latino e storia dell'arte
da bern-1-16-4-13
16 dic 2015, 22:18
Forum: Olimpiadi della matematica
Argomento: Winter Camp 2016
Risposte: 134
Visite : 59551

Re: Winter Camp 2016

Ma fai bene! Uno nella vita deve avere delle priorità
da bern-1-16-4-13
16 dic 2015, 22:15
Forum: Olimpiadi della matematica
Argomento: Winter Camp 2016
Risposte: 134
Visite : 59551

Re: Winter Camp 2016

A me invece non me ne può fregar di meno delle altre materie :lol: :lol: :lol:
Basta non bocciare, ma quello non è un problema...

In teoria potrei anche restare a casa questi ultimi giorni di scuola :idea: :mrgreen: